0 Daumen
1,6k Aufrufe

Hallo :)

Könnte mir einer bei dieser Aufgabe bitte helfen oder zumindest einen Tipp geben? :( Ich weiß nicht wie ich vorgehen soll. 

B45AB56A-3DF5-4B85-B154-6BA9F17C8457.jpeg

Danke ! :)

Avatar von

"Ich weiss nicht wie ich vorgehen soll. "

Du kannst versuchen ein Gegenbeispiel zu basteln. Wenn das nicht gelingt, kannst du immer noch versuchen, die Behauptung zu beweisen. 

Mein Gegenbeispiel
f(x) = sin(x)
g(x) = |sin(x)|
Kontrolliere das mal. Das Intervall kannst du selbst noch festlegen (genügend gross).

Mein Gegenbeispiel enthält selbstverständlich Nullstellen. Geht also nicht. 

Vielleicht so

f(x) = 1/ sin(x)
g(x) = 1/  |sin(x)|   ?

HIer ist das Problem, dass es Definitionslücken gibt und man kein Intervall mehr hat. 

Vielleicht stimmt die Behauptung tatsächlich (?) 

Wie ware es mit Wurzelziehen auf beiden Seiten: f(x)=±g(x).

Vielen Dank ich versuch es mal :)

2 Antworten

+3 Daumen
 
Beste Antwort

Ich versuche es mal durch Widerspruch:

Seien also a,b aus I mit  f(a) = g(a) und  f(b) = - g(b)   #

Betrachte die Differenz d(x) = f(x) - g(x) ist also auch eine stetige Funktion auf I

Da für alle x entweder f(x)=g(x) oder f(x) = - g(x) gilt, hat man:

Für alle x∈I  gilt entweder d(x)=0 oder d(x)= 2*g(x).    ##

Wegen # gilt d(a)=0 und d(b) = 2*g(b) ≠ 0 , da nach Vor.   weder f noch g Nullstellen haben.

Also liegt  g(b) zwischen 0 und 2*g(b).

Also gibt es nach dem Zwischenwertsatz ein c zwischen a und b mit  d(c) = g(b)

Wegen ## und g(b)≠0 gilt    d(c) = 2*g(c)  also  

                                             f(c) - g(c) = 2*g(c) 

                                                 f(c) = 3*g(c)  

im Widerspruch zu  f(c)=g(c)  oder f(c) = - g(c).

Avatar von 287 k 🚀

Dankeschön mathef, habe es bisschen anders gemacht komme aber auf das Selbe :)

+2 Daumen

$$ f(x)^2 - g(x)^2 = [ f(x) - g(x) ] \cdot [ f(x) + g(x) ] = 0 $$ Daraus ergibt sich die Behauptung.

Avatar von 39 k

Brauchst du da noch den Zwischenwertsatz, um das auf ganz I zu verallgemeinern? 

Ich glaube nicht, da die Relation ja auf ganz \( I \) gilt.

Warum genau, ist bereits ausgeschlossen, dass für x1 ≠ x2 innerhalb des Intervalls einmal 

f(x1) - g(x1) = 0

und bei x2 

f(x2) - g(x2) = 0  gilt ? 

Das ist doch wohl gerade der Pfiff. Und das es nicht wechselt hat wohl

mit der Stetigkeit zu tun und kann vermutlich mit dem Zwischenwertsatz

bewiesen werden.  Wie - fällt mir auch nicht ein.

Ein anderes Problem?

Stell deine Frage

Willkommen bei der Mathelounge! Stell deine Frage einfach und kostenlos

x
Made by a lovely community